FoRT practice test (multiple choice only)

¡Supera tus tareas y exámenes ahora con Quizwiz!

88. Which of the following best describes the primary advantage of having a student read a passage silently and then provide a "retelling" as a means of assessing the student's comprehension, rather than having the student answer questions? A. A retelling is open-ended and requires the student to construct a description of the passage more independently of the examiner. B. The results of a retelling are more objective and easier to quantify than the results of direct questioning. C. The procedure involved in retelling tends to be more familiar to a wider range of students, including English Language Learners. D. A retelling can provide information about the student's inferential comprehension skills, which questioning cannot provide.

A. A retelling is open-ended and requires the student to construct a description of the passage more independently of the examiner.

32. Which of the following sentences contains a pair of italicized words that differ from one another by one phoneme? A. He took off his cap so that he could take a nap. B. She works at a bank that is located near the bank of a river. C. She told him not to buy a ticket because she had already bought one. D. His face looked pale after he carried the pail of water for a mile.

A. He took off his cap so that he could take a nap.

21. A kindergarten teacher wants to promote students' understanding of the alphabetic principle. Which of the following would be the most effective first step in a sequence of instruction designed to achieve this goal? A. Talk with students about selected consonants using a series of posters that each feature one consonant and contain pictures of items whose initial phoneme demonstrates that consonant's sound. B. Have students trace both lowercase and uppercase letters of the alphabet and then practice reproducing the letters on their own. C. Talk with students about the title, beginning, middle, and end of a story and point to these parts while reading the story aloud from a big book. D. Put labels on several familiar objects in the classroom and regularly read the labels aloud to the students

A. Talk with students about selected consonants using a series of posters that each feature one consonant and contain pictures of items whose initial phoneme demonstrates that consonant's sound.

18. Which of the following strategies would be most effective in promoting kindergarten children's ability to recognize and name letters of the alphabet? A. The teacher says the name of a letter while the children each trace its shape on a cutout letter. B. The teacher posts the entire alphabet around the room in several different formats. C. The teacher reads aloud to the children from books that contain mostly words that follow regular phonics patterns. D. The teacher emphasizes the initial sounds of words when reading to the children.

A. The teacher says the name of a letter while the children each trace its shape on a cutout letter.

89. Which of the following criteria would be most important to consider when selecting "leveled texts" for use in assessments and guided reading with beginning-level readers? A. The texts should use repeated words and natural oral language structures. B. The texts should require readers to use problem-solving to connect text to illustrations. C. The texts should emphasize use of literary language and dialogues. D. The texts should feature a range of punctuation and context-specific vocabulary.

A. The texts should use repeated words and natural oral language structures.

6. Which of the following first-grade students has attained the highest level of phonemic awareness? A. a student who, after hearing the word hot and the sound /Ʊ/, can substitute /Ʊ/ for /ǂ/ to make the word hit B. a student who can orally segment the word wonderful into won-der-ful C. a student who, after hearing the words fish and fun, can identify that they both begin with the same phoneme, /f/ D. a student who can orally segment the word train into its onset and rime

A. a student who, after hearing the word hot and the sound /Ʊ/, can substitute /Ʊ/ for /ǂ/ to make the word hit

24. A teacher can most effectively support first graders' development of rapid automatic word recognition by first teaching students how to: A. apply consistent phonics generali-zations in common words. B. use context clues to determine the meanings of words. C. identify the constituent parts of multisyllable words. D. look up unfamiliar words in the dictionary.

A. apply consistent phonics generali-zations in common words.

25. A second-grade teacher pairs students who are reading at approximately the same independent reading level for a partner-reading activity. During the activity, the two partners sit side by side and take turns reading aloud from a shared text. Over a period of several days, the partners read a large number of independent-level texts together. This activity is best designed to promote students': A. development of reading rate and automaticity. B. awareness of key aspects of prosodic reading. C. development of comprehension skills and strategies. D. awareness of new phonics elements.

A. development of reading rate and automaticity.

68. Students in a third-grade class are working on an interdisciplinary unit on Native Americans of the Northeast. The teacher has selected a historical novel for students to read during the unit to help them gain insight into people's daily lives in a particular Native American nation at a particular point in time. However, the teacher is aware that the novel's text complexity may make comprehension difficult for a group of struggling readers in the class. Which of the following strategies would be most effective for the teacher to use to support the struggling readers' comprehension of the novel and their purpose for reading? A. engaging the students in guided reading and rereading of key passages in the novel B. having the students stop after reading each chapter and try to summarize the key events of the plot in their own words C. asking the students to rewrite the story from the perspective of a different character in the novel D. encouraging the students to read key chapters of the novel aloud together by taking turns reading specific pages

A. engaging the students in guided reading and rereading of key passages in the novel

22. A second-grade teacher regularly reviews spelling patterns previously taught. The teacher also provides students with multiple opportunities to read and write connected text that features words containing the target spelling patterns and to engage in word sorts focused on previously taught spelling patterns. These types of activities are likely to promote students' reading proficiency primarily by developing their: A. knowledge of grade-level vocabulary. B. reading fluency with respect to accuracy. C. awareness of different types of morphemes. D. word recognition with respect to sight words.

A. knowledge of grade-level vocabulary.

35. Use the information below to answer the question that follows. A teacher poses the following question to fourth-grade students. What words can you think of that have the word "act" in them? Using student responses, the teacher creates the following web on the board. This technique is likely to be most helpful for enhancing the students' awareness of: A. morphemic structure. B. compound words. C. syllable patterns. D. Greek roots.

A. morphemic structure.

4. A teacher holds up a series of familiar objects, asking students to name each object and isolate the final sound they hear. This type of activity would be most appropriate for a student who: A. needs help developing phonemic segmentation skills. B. is performing below grade-level benchmarks in reading fluency. C. lacks automaticity in word recognition. D. has difficulty sounding out phonetically regular one-syllable words.

A. needs help developing phonemic segmentation skills.

73. A sixth-grade class is working on an Internet research project about various natural resources and their uses. The teacher could best support students' effective use of the Internet for their research by: A. providing students with a checklist of questions that prompt critical evaluation of information on Web sites. B. giving students a list of Web sites that have been preapproved based on the sites' reading levels. C. encouraging students to search for Web sites that are easy to navigate and that contain familiar vocabulary. D. teaching students to employ a variety of search engines to locate relevant Web sites.

A. providing students with a checklist of questions that prompt critical evaluation of information on Web sites.

67. Read the passage below; then answer the question that follows. For the second time that week, Saul forgot to wash his hands after working on his painting. He had gotten so involved filling in the ocean in his picture that he had barely even heard the teacher telling everyone it was time to put away their easels and wash up for lunch. He had put his supplies away, but, still thinking about the ocean, he had gone straight to his desk. Now he saw that he was leaving blue-paint handprints on his desk, on his shirt, on his books—even on his lunchbox. Estella looked over at him and joked, "Hey, Saul! You're the new King Midas! Only you turn everything to blue!" Saul rolled his eyes at her as he got back up to go to the sink. This passage would be most suited for helping students: A. recognize a literary allusion. B. analyze story elements. C. predict future events. D. analyze an author's point of view.

A. recognize a literary allusion.

80. The teacher asks students to locate and mark places mentioned in the chapter ona map as they read. This activity is most likely to help students: A. use visualization to facilitate their comprehension of the text. B. paraphrase content to make the text more understandable. C. connect elements in the text to their background knowledge. D. identify the text's main ideas and supporting details.

A. use visualization to facilitate their comprehension of the text.

30. Which of the following statements best describes how oral vocabulary knowledge is related to the process of decoding written words? A. A reader applies decoding skills to unfamiliar written words in order to increase his or her oral vocabulary knowledge. B. A reader's oral vocabulary knowledge allows the reader to derive meaning as he or she decodes written words. C. A reader must have extensive oral vocabulary knowledge in order to learn decoding processes. D. A reader's oral vocabulary knowledge is dependent on his or her development of strong decoding skills.

B. A reader's oral vocabulary knowledge allows the reader to derive meaning as he or she decodes written words.

10. A beginning-level English Language Learner can consistently blend individual phonemes to make simple English words composed of two or three phonemes but is having difficulty blending the sounds of familiar single-syllable words composed of four phonemes (e.g., clip, trap, spin). Which of the following questions would be most important for the first-grade teacher to consider when addressing the needs of this student? A. Are the target words in the student's oral vocabulary in English? B. Does the student's primary language have consonant blends? C. Can the student distinguish between short and long vowel sounds in English? D. Do the target words have cognates in the student's primary language?

B. Does the student's primary language have consonant blends?

69. A teacher can best help sixth graders draw inferences from informational text by asking them to complete which of the following statements? A. In my opinion . . . B. The passage suggests . . . C. In comparison . . . D. The author's first point is . . .

B. The passage suggests . . .

82. Which of the following types of assessments would best provide information about the comparative reading proficiency of students in an elementary school? A. a test of vocabulary development B. a norm-referenced survey test C. a reading miscue inventory D. a diagnostic portfolio

B. a norm-referenced survey test

17. Pointing out the title, beginning, middle, and end of a book to a group of preschool children before reading the book aloud to them contributes to their reading development primarily by promoting their: A. understanding of text directionality. B. development of book-handling skills. C. understanding of the concept of schema. D. development of literal comprehension strategies.

B. development of book-handling skills.

64. A second-grade teacher reads a trade book aloud to the class. Which of the following postreading activities would be most likely to promote the students' comprehension of the story by enhancing their literary analysis skills? A. encouraging the students to identify the key vocabulary words in the story B. discussing with the students how the characters in the story respond to major events and challenges C. asking the students to reread the story silently and respond to several literal comprehension questions D. having the students "freewrite" about the story in their reading response journals

B. discussing with the students how the characters in the story respond to major events and challenges

76. A fifth-grade teacher gives students a reading guide for an informational text that they will be reading independently. The reading guide contains various activities, including prompting students to summarize certain passages, to explain relationships between concepts according to specific information in the text, and to determine the meaning of domain-specific words based on appositives or appositive phrases embedded in the text. This reading guide is likely to be most effective for achieving which of the following instructional purposes? A. developing students' ability to read the text evaluatively B. encouraging students to read and interact closely with the text C. supporting students' development of prosodic reading skills D. teaching students to adjust their reading rate based on text complexity

B. encouraging students to read and interact closely with the text

8. A kindergarten teacher asks a small group of students to repeat after her. First, she says the word grape and then pronounces it as gr and ape. Next, she says the word take and then pronounces it as t and ake. This activity is likely to promote the students' phonemic awareness primarily by: A. helping them recognize distinct syllables in oral language. B. encouraging them to divide words into onsets and rimes. C. teaching them how to distinguish between consonants and vowels. D. promoting their awareness of letter-sound correspondence.

B. encouraging them to divide words into onsets and rimes.

71. A third-grade teacher periodically reads aloud from a chapter in content-area textbooks and describes his thought processes as he reads. Following is an example: "'The moon does not shine on its own. The sun's light reflects off the moon.' Hmm. I'm imagining that the sun is like a flashlight shining on the moon in the dark. 'As the moon rotates, only the part that faces the sun is visible from the Earth.' I'm not quite sure what "visible" means, but it sounds kind of like vision, which I know has to do with eyes. It probably means the part that we can see from the Earth. Now, that makes me wonder— why do we see different amounts of the moon at different times? Let's see if the next part of the chapter explains this . . ." This practice is most likely to promote students' reading proficiency by: A. exposing them to new vocabulary in context. B. modeling for them metacognitive comprehension strategies. C. giving them an example of fluent oral reading. D. summarizing for them the main ideas of an expository text.

B. modeling for them metacognitive comprehension strategies.

9. A teacher shows a student pictures of familiar objects. As the teacher points to the first picture, she asks the student to name the object in the picture. Next, she asks the student to count on his fingers the number of sounds he makes as he says the word again. This activity is most likely to promote which of the following? A. understanding of the alphabetic principle B. phonemic awareness skills C. development of letter-sound correspondence D. word identification skills

B. phonemic awareness skills

72. Skimming is likely to be the most effective strategy for accomplishing which of the following reading tasks? A. evaluating the validity of information on an Internet Web site B. previewing a chapter in a content-area textbook C. synthesizing information from various sources for a research report D. studying specific facts for a content- area exam

B. previewing a chapter in a content-area textbook

87. An advantage of using assessment tools such as portfolios and scoring rubrics is that they: A. provide more objective results than do multiple-choice tests. B. promote student participation in self-assessment activities. C. ensure consistency among different evaluators. D. offer more reliable assessment data

B. promote student participation in self-assessment activities.

2. A kindergarten teacher could best determine if a child has begun to develop phonemic awareness by asking the child to: A. count the number of words the child hears in a sentence as the teacher says the sentence. B. say the word cat, then say the first sound the child hears in the word. C. point to the correct letter on an alphabet chart as the teacher names specific letters. D. listen to the teacher say boat and coat, then identify whether the two words rhyme.

B. say the word cat, then say the first sound the child hears in the word.

3. As students begin to read, the ability to blend phonemes orally contributes to their reading development primarily because it helps students: A. recognize and understand sight words in a text. B. use knowledge of letter-sound correspondence to decode words. C. guess the meaning of unfamiliar words from their context. D. divide written words into onsets and rimes.

B. use knowledge of letter-sound correspondence to decode words.

83. Considerations of validity in test construction relate most closely to: A. how a particular examinee's test performance relates to a preestablished standard. B. whether the test questions effectively measure their specified content. C. how a particular examinee's test performance compares to the performance of other examinees. D. whether the test results are likely to be repeatable with a similar examinee test group.

B. whether the test questions effectively measure their specified content.

85. Which of the following informal assessment results provides the clearest indication that a kindergarten child has attained a beginning level of phonemic awareness? A. The student can clap the "beats" or syllables of familiar multisyllable words. B. The student can delete the second "word" or syllable in compound words. C. The student can identify the beginning sound of single-syllable words. D. The student can substitute phonemes in the medial position of single-syllable words.

C. The student can identify the beginning sound of single-syllable words.

63. Sixth-grade students have just finished reading a chapter in a novel and are getting ready to write an entry in their response journals. The teacher could most effectively develop students' literary response skills by assigning which of the following journal prompts? A. What new vocabulary words did you learn when reading this chapter? List and define the new words from the chapter. B. What happened in the chapter? Describe two or three events from the chapter. C. What do you think is the main idea or theme of the novel? Relate specific events in this chapter to the theme you suggest. D. Which characters are mentioned in this chapter? List each of the characters.

C. What do you think is the main idea or theme of the novel? Relate specific events in this chapter to the theme you suggest.

90. In order to select a trade book that emphasizes predictability, a teacher should ensure that: A. the text includes some pictures or illustrations. B. the concepts in the text are at an appropriate level of difficulty for the target student(s). C. a phrase, rhyme, or sentence is repeated throughout the text. D. the length of the text is not likely to exceed the attention span of the target student(s).

C. a phrase, rhyme, or sentence is repeated throughout the text.

33. Which of the following students demon- strates variation in reading development that would require intervention focused on explicit phonics instruction? A. a kindergarten student who can recite the alphabet from memory but has difficulty distinguishing individual phonemes in words B. a first-grade student who can easily decode nonsense words but has limited comprehension of the meaning of text C. a second-grade student who is adept at using context clues to identify words but has difficulty sounding out the letters in unfamiliar words D. a third-grade student who can read most grade-level text fluently but has difficulty with unfamiliar irregular low-frequency words

C. a second-grade student who is adept at using context clues to identify words but has difficulty sounding out the letters in unfamiliar words

1. Which of the following students is demonstrating the specific type of phonological awareness known as phonemic awareness? A. a student who, after being shown a letter of the alphabet, can orally identify its corresponding sound(s) B. a student who listens to the words sing, ring, fling, and hang and can identify that hang is different C. a student who, after hearing the word hat, can orally identify that it ends with the sound /t/ D. a student who listens to the word magazine and can determine that it contains three syllables

C. a student who, after hearing the word hat, can orally identify that it ends with the sound /t/

65. A fifth-grade class is about to read a play about the life of Harriet Tubman called "Travels on the Railroad." Which of the following prereading activities would best promote students' comprehension of the text? A. introducing the common elements of plays as a genre and looking at sections of a printed play as a class B. asking students to generate several questions about the play based on the play's title C. asking students to share what they already know about the time period during which the play takes place D. encouraging groups of students to create and perform their own short skits about the same subject

C. asking students to share what they already know about the time period during which the play takes place

16. A preschool teacher is reading a story to his class. As he reads, he holds the book so the children can see the words and pictures while his finger follows the line of print. This activity would contribute to the children's reading development primarily by: A. promoting their development of letter recognition skills. B. helping them recognize phonemes that occur frequently in print. C. developing their awareness of left-to-right directionality. D. promoting their understanding of letter-sound correspondence.

C. developing their awareness of left-to-right directionality.

26. Which of the following strategies would be most effective in promoting second graders' decoding of multisyllable words? A. giving students opportunities to read literature that offers repeated exposure to predictable text B. prompting students to sound out the individual phonemes that compose multisyllable words C. encouraging students to compare the parts of new multisyllable words with known single-syllable words D. reinforcing students' recognition of high-frequency multisyllable words using drills and flashcards

C. encouraging students to compare the parts of new multisyllable words with known single-syllable words

84. If a standardized test is said to lack reliability, the test: A. is not measuring what it is supposed to measure. B. has not proven to be useful as an instructional intervention. C. gives fluctuating scores in different administrations. D. has poor predictive value relative to students' classroom performance.

C. gives fluctuating scores in different administrations.

12. A preschool child picks up an unfamiliar book, opens it to the end, points to the text, and begins to "pretend read" the story. These behaviors suggest that the child most likely: A. has well-developed book-handling skills. B. knows where individual words begin and end. C. has developed an understanding that print carries meaning. D. understands the concept of print directionality.

C. has developed an understanding that print carries meaning.

7. Asking students to listen to a word (e.g., same) and then tell the teacher all the sounds in the word is an exercise that would be most appropriate for students who: A. have a relatively low level of phonological awareness. B. are beginning to develop systematic phonics skills. C. have a relatively high level of phonemic awareness. D. are beginning to master the alphabetic principle.

C. have a relatively high level of phonemic awareness.

5. Phonemic awareness contributes most to the development of phonics skills in beginning readers by helping them: A. recognize different ways in which one sound can be represented in print. B. count the number of syllables in a written word. C. identify in spoken language separate sounds that can be mapped to letters. D. understand the concept of a silent letter.

C. identify in spoken language separate sounds that can be mapped to letters.

81. A third-grade teacher has been conducting a series of ongoing assessments of a student's oral reading. Shown below is a sentence from a text, followed by a transcription of a typical example of the student's oral reading performance. Text: Her boots crunched through the snow. Student: Her boats crucked throw the snow. After reading the sentence, the student paused and then reread it without the teacher's prompting and self-corrected the errors. Based on this information, the teacher could best meet this student's needs by adjusting instruction in order to: A. enhance the student's oral vocabulary development. B. develop the student's ability to self-monitor comprehension. C. improve the student's decoding skills. D. promote the student's ability to track print.

C. improve the student's decoding skills.

77. Two proficient readers are answering postreading comprehension questions about a chapter in a content-area textbook. -The first student demonstrates exceptional recall of details from the chapter but has difficulty answering questions about the gist of the chapter. -The second student can give an outstanding summary of the chapter but has difficulty remembering specific facts from the chapter. Which of the following best explains the most likely reason for the students' varied understanding of the text? A. translating textbook reading assignments from English into her primary language B. receiving reading comprehension instruction with texts written in her primary language C. learning to use metacognitive reading strategies with English text D. reading texts in her primary language that cover the same material as her English textbooks

C. learning to use metacognitive reading strategies with English text

78. An English Language Learner reads academic texts fluently in her primary language but is struggling to understand her content-area textbooks in English. This student would likely benefit most from engaging in which of the following activities? A. translating textbook reading assignments from English into her primary language B. receiving reading comprehension instruction with texts written in her primary language C. learning to use metacognitive reading strategies with English text D. reading texts in her primary language that cover the same material as her English textbooks

C. learning to use metacognitive reading strategies with English text

31. Read the sentence below; then answer the question that follows. My family went to the circus last weekend. I liked the clowns the best. They were very funny. A student makes several miscues when reading these sentences aloud. Which of the following miscues represents an error in decoding consonant blends? A. omitting circus B. pronouncing clowns as clones C. saying bet for best D. shortening funny to fun

C. saying bet for best

27. According to basic principles of research- based, systematic phonics instruction, which of the following common English letter combinations would be most appropriate for a first-grade teacher to introduce first? A. ir B. kn C. th D. oi

C. th

A fifth-grade teacher plans to have students read a chapter about the American Revolutionary War from their social studies textbook. The following is an excerpt from the chapter. The Battle of Bunker Hill took place on June 17, 1775. At the time, the American army occupied the area from Cambridge to the Mystic River. American troops gathered in Cambridge Common on the evening of June 16, 1775, and set out for Bunker Hill. Upon reaching Bunker Hill, however, officers decided to move to Breed's Hill, a smaller hill closer to Boston. 79. Based on this excerpt from the chapter, which of the following graphic organizers would best promote students' awareness of the chapter's text structure? A. outline B. Venn diagram C. timeline D. semantic map

C. timeline

Explanation for 67

Correct Response: A. A literary allusion is when an author refers indirectly to an earlier literary work by using a name, word, or phrase closely associated with that literary work. In this passage, the character Estella refers to King Midas, a character of Greek legend who was given the power to turn everything he touched to gold. While a teacher could use this passage to analyze perhaps one story element, such as character (B), or possibly to predict future events (C), the passage is not particularly well suited to teach these skills. Also, in this passage the author renders the action objectively (describing a series of observable events in emotionally neutral language without commentary); thus, it would not lend itself to an analysis of the author's point of view (D).

explanation for 32

Correct Response: A. A phoneme is a phonological unit of language, a discrete speech sound in a particular language that native speakers of the language recognize as sufficient to distinguish between two phonologically similar but separate words. For example, the speech sounds /k/ and /n/ are phonemes in English because native English speakers who hear the pair of spoken forms /kăp/ and /năp/ regard them as distinct words. B is incorrect because the two identically spelled words bank (financial institution) and bank (land form abutting a river) are pronounced the same way; i.e., they contain the identical sequence of phonemes. C is incorrect because the two words differ by more than one phoneme. D is incorrect because although the pair of words, pale and pail, differ in their spelling, they are identical in their pronunciation; i.e., they contain the same phonemes.

explanation for #24

Correct Response: A. Automaticity is the rapid recognition of a word without conscious attention to the decoding process. Research indicates that accurate decoding skills are a prerequisite to the development of, and readiness to benefit from instruction in, automatic word recognition. Applying consistent phonics generalizations to decode common words is a foundational decoding skill appropriate for beginning readers at the first-grade level. B is incorrect because a reliance on context clues for word identification is a frequent cause of inaccurate reading, so this strategy would not facilitate automatic word recognition. C is incorrect because dividing multisyllable words into constituent parts is an advanced skill typically taught after developing readers have achieved automaticity reading many single-syllable words. D is incorrect because looking up words in a dictionary is related to decoding only as a corrective strategy and does not present phonics patterns explicitly or systematically. It is therefore of limited effectiveness in developing accurate decoding skills and automaticity among beginning readers.

Explanation for 89

Correct Response: A. Beginning readers need lots of practice reading a variety of texts written at their independent reading levels to reinforce their knowledge of phonics and sight words already taught, build their automaticity, and improve their reading fluency. Leveled texts are series of texts constructed using controlled vocabulary and syntactic structures. Within a particular level, the texts share many of the same vocabulary words and follow basic syntactic structures, both of which gradually increase in difficulty in successive levels.Matching students to appropriate-level texts is key to providing beginning readers with effective practice opportunities. B is incorrect because the primary goal of using these texts with beginning readers is to reinforce students' decoding skills and promote reading fluency, not to promote their use of picture clues. They are also not designed to promote students' knowledge of literary language or devices (C). Because of their controlled nature, leveled texts feature a limited range of punctuation, and they purposefully introduce only a few new content vocabulary words per text (D).

Explanation for 73

Correct Response: A. Determining whether a given Web site contains reliable information is an important step in conducting research on the Internet. Having students question the validity of content they read on a Web site will build this skill, promoting their ability to conduct more effective and bias-free research. B is incorrect because giving students a list of preapproved Web sites does not promote their independent research skills. Encouraging them to rely only on user-friendly Web sites (C) could lead students to ignore Web sites that contain valuable, objective information. Employing a variety of search engines (D) does not necessarily lead to more effective research, since different search engines are likely to identify a similar list of potential Web sites.

46. Before beginning a new content-area reading passage, a fourth-grade teacher asks students to think of words related to the topic of the text. The teacher writes the words on the board and then asks the students to suggest ways to group the words based on meaningful connections. The teacher also encourages them to explain their reasons for grouping particular words together. This series of activities is likely to promote the students' reading development primarily by helping them: A. extend and reinforce their expressive and receptive vocabularies related to the text's topic. B. infer the meaning of new vocabulary in the text based on word derivations. C. strengthen and extend their understanding of the overall structure of the text. D. verify word meanings in the text by incorporating syntactic and semantic clues into their word analysis.

Correct Response: A. Grouping words related to a text based on conceptual categories and the words' associative meanings helps deepen students' understanding of the vocabulary. Discussing and justifying connections among the words further enhances students' understanding of the words and promotes retention of new words. B is incorrect because, while exploring the etymology of target vocabulary words can also be an effective strategy for deepening students' understanding of the words, the series of activities described in the scenario did not involve exploration of word derivations. Nor did it involve using semantic or syntactic clues from the text to verify words' meanings (D). C is incorrect because studying the conceptual connections among key vocabulary in a text is not likely to enhance students' understanding of the text's structure.

explanation for 68

Correct Response: A. Guiding students in reading and rereading key passages of a complex text scaffolds their understanding of the text by allowing them to delve deeper into the ideas presented in the text with each successive reading. The activities described in responses B, C, and D would not be effective in facilitating struggling readers' comprehension of the novels because these activities do not provide the students with sufficient scaffolding to support students' construction of meaning of a complex text.

explanation for 88

Correct Response: A. Having a young student provide a "retelling" (or having an older student construct a summary) of a text removes clues or scaffolds from the assessment that teacher questions often provide and ensures that a student's responses are based solely on his or her reading of the text. Some disadvantages of retellings include that the results are not easy to quantify (B) and the procedure is often unfamiliar to students, particularly to students who are new to U.S. schools (C). Also, since retellings are unstructured and usually elicit only explicitly stated details about the text, this type of assessment is not necessarily effective in assessing a student's inferential comprehension skills (D).

explanation for #4

Correct Response: A. In the activity described, students are asked to isolate and pronounce separately the final sound, or phoneme, of a familiar word. Learning to isolate the final sound in a word is a step toward mastering phonemic segmentation, an important phonemic awareness skill that supports literacy development in English. B, C, and D are incorrect because they are related to decoding print, which is not addressed in this activity.

62. A fifth-grade teacher guides students in reading a complex literary text. First, the teacher reads aloud the beginning of the text as the students follow along silently in their copies. Next, the teacher rereads key phrases and sentences, asking students what the author meant by certain statements or by the choice of certain words. Finally, the teacher and students reread the section aloud together with expression. The teacher repeats these steps with each section of the text. This activity promotes reading proficiency primarily by: A. modeling for students how to engage in close reading of academic texts. B. developing students' word consciousness and love of interesting new words. C. helping students achieve grade-level fluency benchmarks for accuracy and rate. D. encouraging students to apply metacognitive comprehension strategies as they read.

Correct Response: A. In the scenario, the teacher guides students' engagement in key processes involved in close reading, such as rereading a complex text to enhance comprehension, analyzing the author's use of language, and questioning the author's intentions and goals. B is incorrect because, while the activity does prompt students to think about the author's choice of words in certain sentences, the activity does not "primarily" focus on vocabulary. C is incorrect because the activity focuses more on building students' comprehension, and their prosody by extension, rather than on developing their fluency with respect to accuracy and rate. D is incorrect because the strategies involved in the activity do not necessarily involve metacognition.

explanation for #18

Correct Response: A. Letter naming entails an ability both to distinguish between letters and to associate particular letter shapes with their names. The activity described in A simultaneously activates visual, auditory, kinesthetic, and tactile sensory pathways in learning letter shapes and associating these shapes with their names. Research suggests that such multisensory techniques are effective for this purpose. B, C, and D are incorrect because the activities described do not focus on one or both subskills required in letter naming. The activity described in B exposes students to letter shapes without promoting students' ability to distinguish the letters from one another or associate each letter shape with a letter name. The activities described in C and D do not present students with information about letter shapes or letter names.

Explanation for #2

Correct Response: B. Phonemic awareness, a type of phonological awareness, is the recognition that spoken words are made up of phonemes, the discrete speech sounds of a language. Segmenting the first sound in a spoken word is one of the first phonemic awareness skills to develop and therefore B is an effective informal procedure for assessing phonemic awareness in the beginning stages. A, C, and D are incorrect because performing the tasks described (counting words, recognizing letters and letter names, recognizing rhyming words) does not require phonemic awareness.

explanation for #6

Correct Response: A. Phonemic awareness, the ability to distinguish and manipulate the phonemes in spoken words, is a type of phonological awareness. Reading research indicates that phonological and phonemic awareness skills develop along a continuum from basic to higher-level skills, and that phoneme substitution is a more difficult, or higher-level, skill. Substituting the sound /Ʊ/ for /ǂ/ in the word hot to make the word hit is an example of phoneme substitution. B and D are incorrect because phonemic awareness is not required to perform the phonological awareness skills described. C is incorrect because recognizing alliterative words—words that begin with the same phoneme—is a phonological awareness skill that precedes development of advanced phonemic awareness skills, including phoneme substitution.

Before reading aloud a book about a farm to a group of beginning readers, a first-grade teacher has the students brainstorm and briefly discuss words related to farms. Next, the teacher reads the text aloud from a big book, pointing to each word being read. Periodically, the teacher stops to discuss with students key concepts or events described in the text and to guide students in relating the text to the illustrations. After finishing the read-aloud, the teacher puts the book in the classroom library and encourages the students to read it on their own. 59. The most important reason for putting the book in the classroom library is to promote the students': A. love of reading by facilitating their access to a story that they have already heard, understood, and enjoyed. B. understanding of the alphabetic principle by reinforcing their knowledge of letter-sound correspondences. C. oral language development by providing them with the opportunity to imitate the teacher's reading of a text. D. use of metacognitive strategies by allowing them to practice self-monitoring when reading silently.

Correct Response: A. Reading aloud to children is a research-based strategy for promoting students' interest in reading and in books. Providing students with access to a book that the teacher has already read aloud and discussed with them is likely to foster their love of reading because the teacher has provided various scaffolds for comprehending the story, and thus they are likely to experience some level of success in reading it independently. Responses B, C, and D are incorrect because simply providing access to the book is not likely to contribute to the students' knowledge of the alphabetic principle (B), oral language development (C), or use of metacognitive strategies (D), which all require explicit instruction and guided practice.

56. Over the course of the school year, a sixth- grade student who had been a fluent, proficient reader in previous years is having increasing difficulty comprehending grade-level literary and informational texts assigned in class. The results of informal, curriculum-based assessments indicate that the student still meets grade-level expectations in vocabulary knowledge, but the student's reading rate and comprehension have dropped below grade level. The student also tends to choose fiction and graphic novels written well below the sixth-grade level for independent reading. The student's overall reading performance suggests that the student would likely benefit most from instruction focused on promoting the student's: A. knowledge and skills related to understanding complex academic language. B. understanding of important features of skilled prosodic reading. C. development of decoding skills and automaticity recognizing grade-level sight words. D. skill in applying contextual analysis and other word analysis strategies.

Correct Response: A. Research has shown that independent reading of a wide range of literary and informational texts plays a key role in students' academic language development, as the language of books tends to be more sophisticated than that of everyday social language. The student in this scenario likely has adequate foundational reading skills, given the student's prior reading proficiency. However, the student's independent reading is limited to "fiction and graphic novels written well below" grade level, thus limiting the student's exposure to grade-level academic language, which in turn inhibits progress in reading comprehension. B, C, and D are incorrect because the student had been a fluent, proficient reader in previous years and continues to read low-level texts with pleasure, thus foundational reading skills are not likely at the root of the student's current difficulty with complex academic texts.

explanation for #25

Correct Response: A. Research has shown that rereading the same text several times builds comprehension and improves reading rate and automaticity with respect to the given text, but these gains do not necessarily transfer to other texts. The most effective way for students to improve reading rate and automaticity is to practice reading many texts written at their independent reading level (i.e., texts that they can read accurately). B is incorrect because, while improving one's reading rate may improve one aspect of prosody, it does not necessarily contribute to other aspects of prosody, such as appropriate phrasing and intonation. Clearly, the activity is not "best designed" to promote prosody. C is incorrect because the activity does not focus on developing students' comprehension skills and strategies. D is incorrect because the students are reading independent-level texts (i.e., texts that they can read with very high degrees of decoding accuracy and comprehension). Thus, it is unlikely that the texts contain "new" phonics elements (i.e., elements that have not yet been introduced to these students).

40. The words enjoyable, maneuverable, corruptible, and convertible best illustrate which of the following principles? A. The spelling of a suffix can vary depending on its root word. B. The accented syllable of a root word can shift when certain suffixes are added to it. C. The addition of a suffix can alter the spelling of its root word. D. The pronunciation of a suffix can change when added to certain root words.

Correct Response: A. The four words presented contain the same derivational suffix, which has two variant spellings, -able and -ible. The correct spelling of the suffix in a particular word depends on the root of the word. B, C, and D are incorrect because the phonological and orthographic variations described in these responses do not occur in the words listed.

explanation for 80

Correct Response: A. The passage mentions several sites or landmarks with which students may be unfamiliar. Locating and marking these sites on a map would improve the students' ability to visualize the direction of the troop movements mentioned in the passage. B, C, and D are not skills that would be scaffolded, facilitated, or reinforced by having the students consult maps.

explanation for 35

Correct Response: A. The web shown shows a circle with the base word act in the center and connected to four word lists that are categorized according to different morphological processes—clockwise, adding suffixes or inflectional endings, adding prefixes, adding both, and creating compound words. Thus, the chart illustrates an analysis of morphemes according to type and how these types of morphemes are combined, promoting an understanding of the morphemic structure in words. B is incorrect because only a single compound word is presented in the web, which is insufficient to illustrate and promote understanding of this word type. C is incorrect because syllable patterns are not the focus of analysis. D is incorrect because the only root word in the web, the base word act, is derived from the Latin word actus and is not a Greek root.

Explanation for #3

Correct Response: B. Phonemic blending is the ability to combine a sequence of speech sounds (phonemes) together to form a word. Beginning readers use their skill in phonemic blending and their knowledge of letter-sound correspondence to sound out and blend the sounds of simple printed words. A, C, and D are incorrect because they describe literacy skills that are unrelated to phonemic blending.

explanation for #21

Correct Response: A. Understanding the alphabetic principle entails the recognition that letters and letter patterns used in writing an alphabetic language correspond to the sounds in the spoken words of the language. The activity described in A would promote understanding of the relationship between letters and the initial sounds in familiar words. Focusing on the initial phoneme in these words reflects an understanding that segmenting an initial consonant is a relatively easy phonemic awareness skill that beginning readers have likely mastered and thus is appropriate to use in early instruction on the alphabetic principle. B is incorrect because the activity does not relate letters to the sounds in words. C is incorrect because it does not focus on the letters in a text or relate them to discrete sounds in words. D is incorrect because the individual sounds are not isolated or related to the letters in the printed word.

explanation for 30

Correct Response: B. A reader uses decoding skills to derive the correct pronunciation of a phonically regular word, but the phonological form alone does not prompt understanding of the word's meaning. To understand the word's meaning, the reader must already have the word in her or his oral vocabulary. A is incorrect because decoding an unfamiliar word correctly does not provide the semantic information about the word required to add the word to a reader's oral vocabulary. C is incorrect because decoding is the process of identifying the spoken form of a word based on its printed form, which can be done without knowledge of the word's meaning. D is incorrect because in the early stages of reading development, oral language precedes development of decoding skills.

Explanation for 64

Correct Response: B. At the second-grade level, a significant focus of literary analysis involves thinking about characters and how characters' actions and words provide clues about their motivations. A, C, and D are incorrect because having students identify important vocabulary from the story (A), reread the story and respond to literal comprehension questions (C), or engage in a "freewrite" about the story (D) could enhance their comprehension of the story but would not develop their literary analysis skills.

explanation for #9

Correct Response: B. In the activity described, the student is prompted to say a word and then count the number of sounds, or phonemes, in the word as he pronounces the word again. By focusing attention on the individual component sounds of the word, the student is practicing phonemic segmentation, an important phonemic awareness skill in the continuum of phonological awareness skills. A and C, which concern the recognition of letter-sound relationships, are incorrect because the activity described does not require the student to use letter knowledge. D is incorrect because the activity does not require the student to use word identification skills, which concern decoding printed words.

14. At the end of each school day, a preschool teacher encourages the children to talk about the day's events. As the children describe each event, the teacher writes it on large block paper. Afterward, the teacher reads the list back to the class. This activity would contribute to the children's literacy development primarily by promoting their: A. basic understanding of the alphabetic principle. B. awareness that speech can be represented by writing. C. basic understanding of word boundaries. D. awareness of the relationship between syllables and the spoken word.

Correct Response: B. In the activity described, the teacher demonstrates to preschool children that their spoken utterances can be recorded verbatim in writing and that this written record can later be decoded in spoken form. A is incorrect because the teacher's strategy does not demonstrate relationships between the individual sounds in spoken words and the letters used to represent the words in writing. C is incorrect because the teacher's strategy does not highlight individual words or provide explicit instruction in segmenting sentences into words. D is incorrect because the teacher does not highlight individual syllables or provide explicit instruction in the component syllables of spoken words.

explanation for #8

Correct Response: B. In the activity described, the teacher provides direct instruction in segmenting single-syllable words into onset and rime—that is, into the initial consonant sounds of the word (the onset) and the rest of the word (the rime). Promoting student mastery of onset-rime segmentation prepares students for learning phonemic awareness skills. A is incorrect because the segmenting activity focuses on single-syllable words and does not help students distinguish syllable boundaries. C is incorrect because the segmenting activity does not focus on phonemes, so it does not help students distinguish between different types of phonemes (e.g., vowels and consonants). D is incorrect because the activity is oral and does not present students with information about how the sounds in spoken words relate to the letters in printed words.

explanation for #22

Correct Response: B. In this scenario, the teacher uses both reading and writing activities to reinforce previously taught spelling patterns. Research has established that encoding and decoding are reciprocal processes and that spelling knowledge can contribute to word-reading accuracy. Accuracy is a key component of reading fluency. A is incorrect because the focus of the activity is on word recognition, not vocabulary acquisition. C is incorrect because the activities described do not relate to morphological awareness. Sight words are words that do not follow regular spelling patterns or that contain phonics elements that have not yet been taught, which makes D incorrect.

explanation for 71

Correct Response: B. Metacognitive reading comprehension strategies prompt students to think about their thinking as they read a text. The teacher models two powerful metacognitive strategies: visualizing to support comprehension and self-questioning to clarify understanding and to set a purpose for further reading. A is incorrect because while the teacher talks about the vocabulary word visible and determines what it means deductively, the focus of this portion of the teacher's think-aloud is promoting the students' comprehension of the sentence as a whole in which the word visible occurs. C is incorrect because the teacher pauses several times during reading to make comments about the text, so the teacher does not present an example of fluent oral reading. The teacher could conceivably summarize the text at the end of the think-aloud (D), but there is no evidence in the description to suggest this will occur.

explanation for 82

Correct Response: B. Norm-referenced tests are designed specifically for the purpose of comparing students' performance. Norms are statistics that describe the test performance of a representative sample group. In general, the tests in A and C are designed to measure the degree to which students meet specific objectives (i.e., they are criterion referenced); therefore, A and C are incorrect. A diagnostic portfolio (D) is individually administered and would most likely contain a comprehensive battery of reading assessments designed for diagnostic purposes.

explanation for 76

Correct Response: B. Reading guides such as the one described compel students to reread a complex academic text multiple times so they can complete the activities in the guide, which are specifically designed to help the students interact with the content and language of the text. A is incorrect because the activities do not focus on developing students' evaluative comprehension skills. C is incorrect because use of such a guide would not promote development of prosodic reading, since it is a silent reading activity that does not involve explicit or implicit teacher modeling of or instruction in prosodic reading. A text-based reading guide would not teach students how to adjust their reading rate based on text difficulty (D) because it focuses on just one specific text

explanation for 72

Correct Response: B. Skimming involves a quick, superficial reading of a text to get an overall impression of the material. This would be an appropriate and effective strategy for previewing a textbook chapter. The other responses are incorrect because evaluating the validity of information on a Web site (A), synthesizing information from a variety of sources for research purposes (C), and studying facts for a content-area exam (D) all require a more careful reading of textual materials.

95. As a first-grade teacher reads a big book to a group of students, the teacher points to the beginning consonants of selected words and accentuates the sound the initial letter makes. This activity is most likely to promote the students': A. awareness of multisyllable words. B. ability to isolate individual sounds in words. C. structural analysis skills. D. ability to blend the sounds in words.

Correct Response: B. The activity described contributes to students' beginning reading development in a number of ways, including reinforcing their phonemic awareness (i.e., their ability to distinguish the separate phonemes in a spoken word), their knowledge of letter-sound correspondences, and their understanding of the alphabetic principle. The other responses are incorrect because the activity focuses students' attention on only the first letter of the words, so the activity would not promote students' awareness of multisyllable words (A), their structural analysis skills (C), or their ability to blend all the sounds in a word (D).

explanation for #17

Correct Response: B. The instruction the teacher provides before reading aloud, in conjunction with how the teacher holds and proceeds through the book during reading, helps beginning readers learn how to hold a book with the front cover facing up and the spine on the left, and then how to move through the pages from front to back. A is incorrect because the teacher does not show students lines of text or indicate to them how to track print during reading. C is incorrect because the teacher does not attempt to access students' background knowledge, or schema, related to the text. D is incorrect because the activity does not focus on teaching students strategies for comprehending the text.

Explanation for 69

Correct Response: B. To draw an inference from a text is to derive a conclusion that is not stated explicitly in the text but rather is suggested by facts or premises presented. A is incorrect because stating an opinion about a text does not necessarily involve deducing or deriving a conclusion from explicit information in the text. C and D focus on the use of literal comprehension rather than inferential comprehension.

explanation for 87

Correct Response: B. Typically, the development of student work portfolios involves students in selecting and self-assessing some or all of the materials that are included in their individual portfolios. Students can use scoring rubrics to self-assess and guide their work. The other responses are incorrect because students and teachers can develop their own criteria for selecting student work for a particular portfolio and can develop their own scoring rubrics for evaluating that work; thus, the results of this form of assessment are neither more objective (A), nor more consistent (C), nor more reliable (D) than the results of standardized assessments.

explanation for 83

Correct Response: B. Validity is the degree to which a test measures what it is intended to measure. A is incorrect because criterion referencing is the term used when a student's performance is related to a preestablished standard or set of objectives. C describes norm referencing. D describes test reliability.

explanation for #10

Correct Response: B. While there are approximately 100 speech sounds, or phonemes, used in human language, only a portion of these is used in any given language, and the set varies across languages. Also, each language has its own constraints on permissible phoneme combinations (e.g., consonant clusters, vowel sequences). For example, English allows consonant blends such as [kl], [tr], and [sp] both in initial and medial positions in words, while some languages do not allow them at all or do not allow them in the initial position, which is where they appear in the sample words clip, trap, and spin. English Language Learners frequently have difficulty detecting in a new language phonemes or phoneme combinations that do not occur in their primary language. A and D are incorrect because the student does not need to understand the meaning of a word or know a cognate of the word in the primary language in order to segment or blend the word's phonemes. C is incorrect because the example words contain only short vowel sounds.

38. The following sentence is missing several words. (1) unusual (2) of spices (3) the soup an (4) flavor. A word with the suffix -tion would fit best in which of the blanks in the sentence? A. (1) B. (2) C. (3) D. (4)

Correct Response: B. Words ending with the derivational suffix -tion are nouns (e.g., combination, addition). The syntax of English determines the word orders that are possible (i.e., grammatically correct) in sentences. For example, articles and adjectives precede the nouns they modify. In the sentence, unusual is an adjective, so blank 1 would most likely represent an article (e.g., the, an), and blank 2 would most likely represent a noun. Therefore, blank 2 is an appropriate placement for a word ending in -tion. A is incorrect because blank 1 most likely represents an article. C is incorrect because it is the only likely placement for the verb in this sentence, given the structure of the sentence frame. D is incorrect because blank 4 follows an article and precedes a noun, so it is not a likely position for another noun.

explanation for 31

Correct Response: C. A consonant blend is a sequence of two or more consonants in a word, each of which represents a separate phoneme. For example, the sequence of consonants at the end of the word best represents the sequence of phonemes /s/ and /t/. A student who says bet for best is omitting the letter s, an error in decoding the consonant blend at the end of the word. The other responses are incorrect because these miscues are unrelated to decoding consonant blends. A is a whole-word omission. B represents an error decoding a diphthong. D is an omission of an entire syllable.

explanation for 33

Correct Response: C. A second-grade student who lacks the fundamental skill of sounding out letters has phonics skills well below grade level and is at significant risk for reading failure. Thus, an appropriate intervention for this student would be explicit phonics instruction. A is incorrect because this student has not yet developed phonemic awareness, so an intervention in explicit phonics would be too advanced. B is incorrect because the student's ability to decode nonsense words demonstrates development of phonics skills. D is incorrect because a student who has attained fluency reads accurately and with comprehension, which is overwhelming evidence that the student has accurate decoding skills—including strong phonics skills.

explanation for 65

Correct Response: C. Activating students' prior knowledge related to a text is a powerful strategy for promoting their comprehension of the text. A is incorrect because it develops students' general knowledge of the genre, but it does not facilitate the students' comprehension of the text, which is based on historical events with which they may be unfamiliar. Similarly, asking questions about a text based on its title (B) can be very helpful in setting a purpose for reading and thus supporting reading comprehension; however, the title of this particular play is metaphorical, so it may generate questions that are totally irrelevant. Creating original skits based on the same subject (D) also can extend students' comprehension of a text, but this strategy would be most effective as a postreading activity rather than as a prereading activity.

Explanation for 63

Correct Response: C. At the secondary level, formal literary response involves developing a thesis and providing evidence from the text to support the thesis. To help sixth graders learn how to construct an effective literary response to a text, the teacher should provide opportunities, such as prompted writing in response journals, to develop a thesis related to a text (e.g., identifying a main idea or theme of the text), and then to support their thesis by citing evidence from the text. A emphasizes vocabulary development, while the questions in responses B and D focus students on literal comprehension skills only—that is, identifying factual information explicitly stated in the text.

explanation for #16

Correct Response: C. By following the line of print with his finger while reading, the teacher provides students with a visual demonstration that print is read from left to right. A, B, and D are incorrect because the teacher does not explicitly identify or indicate component letters, phonemes, or letter-sound correspondences related to the text.

explanation for #26

Correct Response: C. By the second grade, students have typically learned to read a wide variety of syllable patterns in single-syllable words. Since most of the syllables in multisyllable words follow the same patterns as those in single-syllable words, the primary challenge for students just learning to decode multisyllable words is learning to recognize the words as a series of discrete syllables. This recognition allows students to apply their prior knowledge of syllable patterns to decoding longer words. The strategy described in C is effective because it focuses students' attention on recognizing the component syllables in these words. A is incorrect because reading predictable text is typically used with beginning readers and is not appropriate for teaching decoding of multisyllable words. B is incorrect because sounding out words letter by letter is a strategy beginning readers can use to decode very simple words with one-to-one letter-sound correspondences, but it is not an efficient strategy for decoding the complex multiletter phonics patterns typically encountered by second-grade readers. D is incorrect because practicing with flashcards—focusing on rapid recognition of words—would not be appropriate until students have learned to process all of the letters in target words systematically and can decode the words with a high degree of accuracy.

60. Which of the following strategies would be most appropriate to use to promote second-grade students' ability to analyze key ideas and details in a literary text? A. explicitly teaching students the key features and conventions of different literary genres B. prompting students to evaluate the significance of a story's setting with respect to its theme C. helping students create a story map of the main characters in a story and the events with which they are involved D. encouraging students to clarify their understanding of a story by reflecting on their personal experiences

Correct Response: C. Characters and plot are key elements of story structure in narrative texts. At the second-grade level, students' literary analysis focuses primarily on these elements. Helping students create a story map focused on main characters and events would both reinforce their understanding of these key components of story grammar and support their analysis of key ideas and details of the text. A is incorrect because learning about the features of different literary genres helps support reading comprehension but it would not necessarily promote students' ability to analyze key ideas and details in a particular text. B is incorrect because, while setting can be important to a narrative text, characters and plot are generally more significant, especially with primary-grades texts. D is incorrect because it focuses more on students' personal responses to a text rather than on their development of literary analysis skills.

explanation for #5

Correct Response: C. English is an alphabetic language—that is, a language in which the letters and letter patterns in written words can be mapped to the phonemes of the spoken words. Phonemic awareness, the recognition of the phonemes in spoken words, and the ability to segment and blend phonemes are critical to learning to apply knowledge of these letter patterns and letter-sound correspondences (i.e., phonics knowledge) to decode and encode printed words. A and D are incorrect because they describe characteristics of phonics that are not related to phonemic awareness. B is incorrect because counting syllables in a written word does not require phonemic awareness.

explanation for 27

Correct Response: C. In research-based, systematic phonics instruction, phonics elements are introduced according to their utility for beginning readers, and therefore according to their frequency of use in words appearing in primary-grades texts. Among the letter combinations given, th occurs most frequently in such texts and therefore is among the very first letter combinations taught. A, B, and D are incorrect because these letter combinations appear significantly less frequently in primary-grades texts than th does.

34. A student who has mastered which of the following skills along the phonological awareness continuum is best prepared to begin explicit phonics instruction? A. being aware that a word is made up of one or more phonemes B. being able to separate a word's onset and rime C. being aware that words can be divided into syllables D. being able to segment and blend a word's phonemes

D. being able to segment and blend a word's phonemes

explanation for 85

Correct Response: C. Phonemic awareness is a specific type of phonological awareness involving the ability to distinguish the separate phonemes in a spoken word. Identifying the beginning sound of a single-syllable word is typically one of the earliest phonemic awareness skills developed, while substituting medial phonemes (D) is considered a more advanced phonemic awareness skill. A and B describe skills at the syllable or word level and are therefore not considered phonemic awareness skills.

Explanation for #1

Correct Response: C. Phonemic awareness is the recognition that spoken words are made up of phonemes—the discrete speech sounds of a language. Identifying the final sound (or phoneme) in a word (C) demonstrates phonemic awareness. A, B, and D are incorrect because the skills described (identifying the sound corresponding to a letter, recognizing rhyming words, and counting syllables in a word, respectively) can be performed without the ability to distinguish separate phonemes in a spoken word.

explanation for 90

Correct Response: C. Reading aloud predictable texts to prealphabetic and emergent readers is an effective way to promote young children's motivation to read and their development of concepts of print. The portion of the text that is predictable is generally a phrase, rhyme, or sentence that is repeated throughout the text, much like the refrain of a song. This repeated text is easy for young children to learn quickly, which allows them to "read" along with the teacher. A, B, and D all describe other important features common to books written for young children. However, these are not features that make the books predictable.

explanation for 84

Correct Response: C. Reliability indicates the degree to which a test yields consistent results over successive administrations. If a test yielded fluctuating results, it would be considered to have low reliability. A relates to a test's content validity, while D relates to a test's predictive validity. Standardized tests are not intended to be used as interventions (B).

45. A sixth-grade student encounters the following sentence in a short story: She experienced a sense of déjà vu as she walked down the street of the strange new city. The student asks the teacher about the meaning of déjà vu in the sentence. The teacher could best respond by advising the student to take which of the following steps? A. Make note of the word in a vocabulary log, and then study the word after finishing the story. B. Use context clues in the sentence to guess the meaning of the word, and then try out that meaning in the sentence. C. Look up the word in the dictionary, and then paraphrase the sentence using the dictionary definition. D. Break the word into its component parts, and then compare the parts to the meanings of similar known words.

Correct Response: C. Stopping to consult a dictionary is generally disruptive to the reading process; however, in some situations, as with the sentence shown in the box, consulting a dictionary is the only effective method for determining the meaning of an unfamiliar term encountered in a text. Also, paraphrasing the sentence by substituting the dictionary definition for the unfamiliar term is a good method for both reinforcing understanding of the new term and returning the reader to the flow of the story. The other strategies given would not be effective in this particular situation. B is incorrect because contextual analysis would not be a useful strategy here, since the sentence around the term déjà vu provides little or no clues as to the term's meaning. Using structural analysis (D) would also not be helpful, since the term is made up of two words and they are borrowed from the French language. Finally, simply noting the term in a vocabulary log and coming back to it after finishing reading the story (A) would cause the student to miss the significance of the sentence and could diminish the student's comprehension of the story.

explanation for #12

Correct Response: C. The child's behavior, "pretend reading" when pointing to the text or printed portion of a page in a book, indicates an understanding that printed text represents meaningful language. A is incorrect because the child begins reading from the end of the book, which indicates a lack of basic book-handling skills. B is incorrect because the child does not point to or decode the separate words of the printed text but only "pretend reads" the story. D is incorrect because the child does not use gestures to demonstrate knowledge of print directionality.

explanation for #7

Correct Response: C. The procedure described—presenting students with a spoken word and having them say all the sounds in the word—is an example of a phoneme-segmentation task. Reading research indicates that phonological and phonemic awareness skills develop along a continuum from basic to higher-level skills, and effective instruction targets skills at a student's current level of development. Segmenting phonemes is a relatively high-level phonemic awareness skill; thus, this exercise would be most appropriate for students who have already achieved a relatively high level of phonemic awareness (C). For this reason, A is incorrect. B and D are incorrect because these responses describe skills at later stages of literacy development.

explanation for 77

Correct Response: C. The student reads academic texts well in her primary language, so she likely has both adequate background knowledge in the content areas and basic reading skills. The student would benefit from using metacognitive strategies such as self-monitoring to connect what she has already learned in the primary language with the content presented in the English textbooks. A, B, and D all focus on primary-language materials. While these strategies may be helpful in supporting the students' ongoing conceptual development with regard to specific content, they do not promote the students' ability to comprehend content-area textbooks in English.

explanation for 78

Correct Response: C. The student reads academic texts well in her primary language, so she likely has both adequate background knowledge in the content areas and basic reading skills. The student would benefit from using metacognitive strategies such as self-monitoring to connect what she has already learned in the primary language with the content presented in the English textbooks. A, B, and D all focus on primary-language materials. While these strategies may be helpful in supporting the students' ongoing conceptual development with regard to specific content, they do not promote the students' ability to comprehend content-area textbooks in English.

66. A second-grade teacher notices that one of her students lacks fluency when reading aloud. The first thing the teacher should do in order to help this student is assess whether the student also has difficulties with: A. predicting. B. inferring. C. metacognition. D. decoding.

D. decoding.

explanation for 81

Correct Response: C. The student's oral reading performance in this sample strongly suggests a lack of foundational knowledge in phonics and sight words. The miscues indicate serious decoding difficulties with various phonics elements, including lack of automaticity in decoding common vowel digraphs (reading boats for boots), common consonant digraphs (reading ck for ch), and complex consonant clusters (reading crucked for crunched). The student also misread a high-frequency sight word (through) that should have been mastered by the end of second grade. Conversely, this oral reading sample provides no evidence that the student has difficulty tracking print (D). And, by self-correcting his or her errors without prompting, the student demonstrates both understanding of the text's vocabulary (A) and the ability to self-monitor for comprehension (B).

Explanation for 79

Correct Response: C. The text structure of this passage is chronological, focusing on when troop movements took place leading up to the battle. Timelines are the most effective form of graphic organizer for conveying a chronology of events. Outlines (A) and semantic maps (D) are well suited to passages that are organized into different sections or categories of related content. Venn diagrams (B) visually display similarities and differences between two or more related topics.

39. Which of the following principles is best illustrated by the words watched, wanted, and warned ? A. Spelling is often the best predictor of the pronunciation of a suffix. B. Open syllables are usually pronounced with a long vowel sound. C. The spelling of a suffix is often more reliable than its pronunciation. D. The second letter of a consonant blend is usually pronounced as the onset of the following syllable.

Correct Response: C. The words listed all contain the regular past-tense inflection -ed. The ending is spelled the same way in all three cases, but the ending is pronounced differently in each word. In the word watched, the -ed ending is pronounced [t]. In the word wanted, the -ed ending is pronounced [Ɵd]. In the word warned, the -ed ending is pronounced [d]. A is incorrect because the three words illustrate that a single spelling of the ending can be pronounced three different ways. B is incorrect because none of the words contain an open syllable and thus do not illustrate characteristics of open syllables. D is incorrect because the syllabication guideline given is relevant to only one of the words (the word wanted).

explanation for 70

Correct Response: D. A persuasive essay is an expository text in which an author takes a stand on an issue and tries to persuade readers by presenting an argument and evidence to support that stand. The teacher's questions are designed to help students analyze the authors' points of view by identifying an author's opinion as presented in an argument, evaluating how effectively the author supports the argument, and considering factors that may have influenced the author's opinion. A is incorrect because, while students may monitor their comprehension of the texts, comprehension monitoring is not explicitly taught or reinforced by the questions. Similarly, the skills described in B and C may be secondary outcomes of this activity but are not the primary goal.

explanation for #11

Correct Response: D. An analysis of the "typical" spelling errors from the student's everyday writing reveals that the student transposes letters in two words (ir in girl, the second nd in independent), omits letters in two words (s in instead, p in interrupted), and repeats letters in one word (ed in decided). These types of errors, coupled with the information that the student performs very well on formal spelling assessments of the same words, suggests that the student overrelies on memorization for spelling. Thus, when spelling words in everyday contexts, the student may not pay attention to the sounds and sound sequences of words, or perhaps the student's phonemic segmentation skills are not automatic. Neither the structure of the sample words nor the examples of the student's spelling errors provide evidence that the student has difficulties with detecting syllable boundaries (A), blending letter-sounds to make words (B), or discriminating between a word's root morpheme and affixes (C).

61. After students in a sixth-grade class finish reading a historical novel about the U.S. Civil War, the teacher asks each student to bring in an object, or a picture or illustration of an object, that, to them, represents the book. The students must also identify a passage or passages from the book that they can use to support their choices when they present their objects to the class. This activity is most likely to promote students' reading development by helping them understand the importance of: A. determining an author's stated or implied main point of view. B. using text structure to develop a general summary of a literary work. C. identifying a novel's mood by analyzing the author's use of figurative language. D. basing interpretations about a literary work on textual evidence.

Correct Response: D. By asking students to think about an object that represents the novel to them and to identify passages from the text to support their choice, the teacher prompts the students to reflect on ideas and/or feelings the book evoked in them as they read and to ensure that their reflections are in fact derived from events in the text and not from their own experience or imagination. A is incorrect because the assignment is open-ended—individual students' interpretations of the book and their choice of an object are unique, so the activity does not require the students to focus on the author's point of view. It also does not necessarily focus their attention on the plot's sequence of events (B). C is incorrect because the activity does not explicitly require students to explore the author's use of figurative language.

explanation for #13

Correct Response: D. By pointing to the letter-like scribbles when reporting to the teacher what the page "says"—the verbal message encoded on the page—the child distinguishes between print and other graphic material. This suggests that the child understands that the function of print is to encode an utterance that can be decoded as meaningful speech. B is incorrect because the description does not indicate that the child has developed knowledge of individual speech sounds in words, that is, phonemic awareness. Thus A and C are incorrect because phonemic awareness is a fundamental prerequisite for understanding the alphabetic principle and the concept of letter-sound correspondences.

20. A preschool teacher shows a group of children pictures of everyday objects. Below each picture is printed the letter of the alphabet that corresponds to the word's initial sound. As the teacher points to each picture, she names the object, then she points to the letter underneath it and says the sound it makes. The teacher invites the children to repeat the sound with her. This activity is likely to contribute to the children's reading development primarily by: A. illustrating the concept of word boundaries. B. focusing on auditory discrimination skills. C. introducing the concept of onset and rime. D. demonstrating that phonemes are represented by letters.

D. demonstrating that phonemes are represented by letters.

explanation for 36

Correct Response: D. In the context of reading, structural analysis is the process of recognizing the morphemic structure of words. Typically, structural analysis is introduced to students in the early elementary grades as a strategy for identifying words with inflections or simple affixes that are in students' oral vocabulary. The list in D features a phonically regular base word and affixes that are appropriate for beginning readers. A is incorrect because the words in this list contain only one morpheme and thus are not useful for demonstrating structural analysis. B is incorrect because the list features words that contain prefixes and roots that are inappropriate for beginning instruction in structural analysis. C is incorrect because this list features a base word (swim) with irregular past tense forms (swam, swum) which do not lend themselves to structural analysis.

explanation for #20

Correct Response: D. In the series of steps described (i.e., pronouncing a word, pointing to the letter that represents the initial sound of the word, and saying the sound the letter makes), the teacher demonstrates that familiar words have component sounds that can be pronounced in isolation, and that letters of the alphabet represent the component sounds (phonemes) of spoken words in print. A is incorrect because the teacher points out neither spoken nor written word boundaries in this lesson. B is incorrect because auditory discrimination entails comparing and distinguishing between two different sounds, whereas only a single sound is presented at a time in this lesson. C is incorrect because the teacher points out only the initial sound of each word and does not point out or discuss the remaining portion (the rime) of the word.

explanation for #15

Correct Response: D. Print awareness encompasses a developing understanding of print concepts and the writing system, an understanding of relationships between oral language and print, and familiarity with ways that literate adults interact with and make use of printed materials and writing. The classroom described includes several elements that can be effectively used to promote print awareness at the kindergarten level. The presence of printed labels, posters, and big books provides exposure to print and the teacher can use these materials to model print functions. Big books can be used effectively as part of explicit instruction in print concepts and book-handling skills. A is incorrect because phonemic awareness—recognition that words are composed of phonemes—is not directly promoted by a print-rich environment. B and C are incorrect because simply exposing children to print does not directly promote word recognition skills.

Explanation for 66

Correct Response: D. Reading fluency is integral to reading comprehension. The key indicators of reading fluency are accuracy, rate, and prosody. In the primary grades, the most common factor disrupting fluency is weak decoding skills, which most directly affects reading accuracy but affects the other indicators as well (e.g., by causing a slow rate or resulting in choppy reading). A, B, and C are incorrect because predicting, inferring, and metacognition are skills or strategies that support reading comprehension. Having difficulty in these skills or strategies would not necessarily affect a student's reading fluency.

explanation for #23

Correct Response: D. Reading research has shown that accurate decoding skills are a prerequisite to effective development of word-reading automaticity, which, in turn, is foundational to the development of reading fluency—that is, reading text accurately, at a rate that supports comprehension, and with speech-like phrasing and expressiveness. Furthermore, convergent research suggests that a lack of automatic decoding skills is a frequent cause of comprehension difficulties among students in the primary grades. A is incorrect because research indicates that poor decoding skills hamper comprehension and the development of reading comprehension skills. B and C are incorrect because being able to decode a text is foundational to understanding the text's meaning.

explanation for 75.

Correct Response: D. Research has shown that fluent readers have higher levels of comprehension than readers who lack fluency. By the third grade, fluent readers have developed automaticity in decoding, which allows them to focus on the meaning of what they are reading rather than on expending all their effort and energy on decoding each individual word letter by letter. Achieving reading fluency does not necessarily mean that students know how to use metacognitive skills (A) or that they have already developed background knowledge at the level of a textbook (B). Having fluency also does not mean that a third-grade student is skilled enough to decode any level of text (C).

explanation for 29

Correct Response: D. Research has shown that the development of spelling and phonics skills is reciprocal; when instruction in these two skills is aligned, development of each skill area is reinforced and enhanced. Thus, learning the spelling patterns of words reinforces learning of their phonics patterns and supports word recognition. A is incorrect because phonemic awareness and the phonological awareness skill of onset-rime segmentation precedes and contributes to students' spelling development, not the reverse as stated in A. B is incorrect because most words targeted in spelling instruction are not new to students but already part of their oral vocabulary. C is incorrect because spelling knowledge continually expands the range of students' decoding skills and does not limit or simplify the decoding process.

explanation for 34

Correct Response: D. Research indicates that phonemic awareness skills, particularly phonemic blending (i.e., combining a sequence of speech sounds to form a word), are prerequisite skills to effective phonics instruction. A is incorrect because the skill described indicates phonemic awareness in the early stages, and not the fully developed skills that are prerequisite for instruction in phonics skills. B and C are incorrect because the phonological awareness skills described develop before the development of phonemic awareness, which is a prerequisite to phonics instruction.

explanation for 86

Correct Response: D. Standardized Informal Reading Inventories (IRIs) are administered individually to students to establish the students' reading levels. The results of this type of assessment provide evidence to guide the selection of reading materials for students for instruction and/or interventions and to guide students in their selection of materials for independent reading. A is incorrect because the results of IRIs do not provide information about how a text's organization or a student's prior knowledge affect the student's comprehension. IRI results are also not helpful in determining what types of clues students use to determine the meaning of unfamiliar vocabulary (B) or in analyzing the relationship between a student's silent- and oral-reading proficiencies (C).

explanation for #19

Correct Response: D. The activity described has children use arm movements and highly textured material to heighten their awareness of letter shapes and the sequence of strokes for forming letters. The activity is effective for students who are having difficulty with letter formation because it simultaneously activates visual, kinesthetic, and tactile sensory pathways in learning letter shapes and associating these shapes with their names. Research suggests that such multisensory techniques are effective for this purpose. A is incorrect because this activity does not address the relationship between the alphabetic writing system of English and the sounds of spoken language. B is incorrect because the function of print is not addressed in this activity. C is incorrect because oral language is not addressed in this activity.

explanation for 91

Correct Response: D. Vocabulary and background knowledge are critical components of reading comprehension, providing the foundation of the schema a student uses to construct meaning from the text. An English Language Learner's English vocabulary is likely to have gaps, particularly with respect to content-area topics that are not typical subjects of everyday conversations. An English Language Learner may be familiar with a content-area topic, but he or she may lack the relevant English vocabulary needed to comprehend an English- language text about that topic. In the scenario described, word analysis and decoding skills are not likely the cause of the student's difficulty (A), since the English Language Learner is able to read with grade-level accuracy. For the same reason, a learning disability that affects language processing (B) and the student's level of first- language literacy (C) are not likely factors impeding his or her reading comprehension.

91. A fourth-grade English Language Learner is new to a school. Assessments suggest that the student can read orally with accuracy on grade level; however, the student's comprehension of grade-level textbooks fluctuates widely. Which of the following steps would be most appropriate for the teacher to take first in order to determine the cause of the student's difficulty? A. Assess the student's word analysis and decoding skills. B. Determine whether the student has a specific learning disability that affects language processing. C. Assess the student's level of first- language literacy. D. Determine whether the student has adequate vocabulary and background knowledge to support comprehension of the textbooks.

D. Determine whether the student has adequate vocabulary and background knowledge to support comprehension of the textbooks.

23. Which of the following best describes the relationship between word decoding and reading comprehension in a beginning reader's development? A. Decoding skills and reading comprehension skills tend to develop independently of one another. B. Reading comprehension skills directly facilitate the development of decoding skills. C. Development of decoding skills is secondary to the development of reading fluency and comprehension skills. D. Rapid automatic decoding skills help facilitate development of reading fluency and comprehension.

D. Rapid automatic decoding skills help facilitate development of reading fluency and comprehension.

29. Which of the following provides the best rationale for incorporating spelling instruction into a first-grade reading program? A. Spelling promotes phonemic awareness by teaching students to break words into onsets and rimes. B. Spelling facilitates vocabulary development by introducing students to new words. C. Spelling simplifies the reading process by focusing students on a limited set of decoding rules. D. Spelling supports word recognition by helping students learn and retain common phonics patterns.

D. Spelling supports word recognition by helping students learn and retain common phonics patterns.

70. A sixth-grade teacher gives students several persuasive essays that present contrasting opinions on a current social issue. The teacher then asks students to consider the following questions as they read the texts. 1. What is the author's opinion on the issue? 2. How might the author's background influence his or her opinion? 3. What evidence does the author use to support his or her opinion? These questions are likely to be most effective for helping students: A. monitor comprehension of informational texts. B. identify the theme in expository texts. C. draw inferences from informational texts. D. analyze point of view in expository texts.

D. analyze point of view in expository texts.

75. A third-grade teacher observes that students who read aloud fluently also demonstrate greater comprehension of expository texts. The best explanation for this is that fluent readers: A. possess a self-awareness that allows them to use metacognitive skills efficiently. B. have already developed the base of background knowledge typically covered by textbooks. C. have well-developed skills for decoding any level of text word by word. D. are able to focus their full attention and cognitive resources on the meaning of a text.

D. are able to focus their full attention and cognitive resources on the meaning of a text.

19. Having kindergarten children practice tracing the letters of the alphabet in sand is most appropriate for children who are having difficulty: A. internalizing the alphabetic principle. B. recognizing that print carries meaning. C. understanding the relationship between spoken and written language. D. developing letter formation skills.

D. developing letter formation skills.

15. A kindergarten teach er hangs labels on key objects in the classroom, puts up posters that include words and captions, and always has a big book on display for the children's use. This kind of classroom environment is most likely to help promote children's: A. recognition that words are composed of separate sounds. B. recognition of high-frequency sight words. C. development of automaticity in word recognition. D. development of an awareness of print.

D. development of an awareness of print.

13. A preschool child draws a stick figure and makes some unintelligible scribbles around it. When she shows it to her teacher, she points to the scribbles and says, "This says 'I love mommy.'" This behavior suggests that the child most likely: A. is ready to learn the concept of letter-sound correspondence. B. is beginning to develop awareness that words are made of distinct phonemes. C. has a basic understanding of the alphabetic principle. D. has grasped the idea that the function of print is distinct from that of pictures.

D. has grasped the idea that the function of print is distinct from that of pictures.

36. Which of the following sets of words would be most effective to use when introducing students to the concept of structural analysis? A. late, great, wait, eight B. afraid, obtain, explain, remain C. swim, swims, swam, swum D. pretest, retest, tested, testing

D. pretest, retest, tested, testing

86. One of the most important purposes of a standardized Informal Reading Inventory (IRI) is: A. to establish how prior knowledge and text organization influence a student's reading comprehension. B. to determine how a student uses semantic, syntactic, and other text clues to deduce a word's meaning. C. to analyze how a student's silent reading comprehension is influenced by oral reading fluency. D. to establish a student's independent, instructional, and frustration reading levels.

D. to establish a student's independent, instructional, and frustration reading levels.

11. A fourth-grade student reads on grade level and consistently scores very high on spelling tests that are part of weekly word study activities. However, the student often misspells the same words, and other familiar words, in everyday writings. The following table shows examples of typical errors the student makes on class writing assignments and in informal notes to friends. Target Word: girl instead decided independent interrupted Student spelling gril intead decideded indepednent interruted

The student's overall spelling performance suggests that the student most likely has a weakness in which of the following foundational skills? A. detecting syllable boundaries in words B. sounding out and blending letter-sounds to make words C. discriminating between a word's root morpheme and affixes D. segmenting and sequencing phonemes in words correct answer: D


Conjuntos de estudio relacionados

Chapter 08 - Understanding Human Sexuality - Hyde - study guide

View Set

Essentials of Leadership Chapter 8 Quiz

View Set